• Treatid
    22
    Weak Solipsism

    The underlying observation of Solipsism is that we only ever experience Sensory Data.

    The weak interpretation of Solipsism is that we cannot prove that our interpretation of indirect observations through Sensory Data accurately reflect what is being indirectly observed.

    You - Sensory Data - Objective Universe (as interpreted from Sensory Data)

    We never have direct experience of an Objective Universe. The existence of an Objective Universe is inferred from our direct experience of Sensory Data. The indirectness of this inference introduces an element of doubt that can never be completely eliminated.

    Strong Solipsism

    We only ever experience Sensory Data. We cannot experience anything that is not Sensory Data.

    Given that an Objective Universe is not Sensory Data we cannot experience an Objective Universe by any means (including indirect inference from Sensory Data).

    Mistaken Assumption

    The existence of an Objective Universe is a mistaken assumption that leads to the above solipsistic interpretations.

    A more accurate assumption would be:

    You - Sensory Data - (currently indirect) Sensory Data.

    Here we are still able to infer things about the wider world from immediate Sensory Data but the wider world is composed of Sensory Data that we aren't directly sensing at this moment.

    {There is an implied distinction between "Objective Universe" and "Sensory Data Universe" that makes this argument significant.}

    Summary

    If we accept the observation that we only ever experience Sensory Data; then any reference to interpretations that include non-Sensory-Data must, in fact, be to Sensory Data.

    non-Sensory-Data is a non-sequitur. We never experience non-Sensory-Data.

    If we experience something then is must have been Sensory Data.

    This isn't the result of word games. The observations that underlie solipsism are not a matter of semantics. We observe that we do not experience anything that isn't Sensory Data.

    This observation isn't conditional. 'indirect' isn't a sufficient modifier to turn non-Sensory-Data into Sensory Data. It is either Sensory Data or it isn't.
  • unenlightened
    8.9k
    If we accept the observation that we only ever experience Sensory Data;Treatid

    I never experience sensory data. It is a mere abstraction that imagines the smell of coffee to be "the same" as birdsong and "the same" as a bunch of words including "Sensory Data".
  • jkop
    721


    Noone observes solipsism, there's no available sensory data of solipsism to encounter, because a solipsist doesn't publish.

    The assumption that we never experience objects and states of affairs, only our own sensory data, is based on a long tradition of bad philosophy of perception refuted by more recent philosophers, e.g. Searle.
  • SophistiCat
    2.2k
    This isn't the result of word games. The observations that underlie solipsism are not a matter of semantics. We observe that we do not experience anything that isn't Sensory Data.Treatid

    Of course these are word games. An experience is an experience of something "in the world" - of being warm or cold, of sitting or standing, of conversing or looking out the window. This is just what the word 'experience' means. We do not experience any such thing as "sensory data" - that is a distortion, poor word games.
  • Philosophim
    2.4k
    I'm not quite sure you understand what solipsism is. Solipsism is the idea that the self is the only thing which can be known to exist. Lets take your idea and say, "Everything is sensory data." Well that includes the self. Meaning we wouldn't know if the self-exists "apart from sensory data" as well. There would be no reason then to say, "I only know I exist."
  • T Clark
    13.1k

    I'm with @Philosophim, I don't think what you are talking about is properly called solipsism. I think it's more a kind of idealism.

    The existence of an Objective Universe is a mistaken assumption that leads to the above solipsistic interpretations.Treatid

    I don't think it's correct to describe the idea of an objective universe as mistaken. What it really is is metaphysical, what R.G. Collingwood calls an "absolute presupposition" or assumption. Collingwood goes on to say that metaphysical positions aren't true or false, right or wrong. In my words, they are points of view, ways of thinking about things, that are more or less useful at particular times in particular situations. That being said, I think the idea that there is no objective reality can be a very useful way of thinking about the world and our experience of it. I say that as someone who came from science, engineering, and materialism.
  • Fire Ologist
    381
    Strong Solipsism

    We only ever experience Sensory Data. We cannot experience anything that is not Sensory Data.
    Treatid

    How can we call it “sensory” then? It’s unsourced data that we call “sensory, but not by any causal connection to anything in particular).

    Sense incorporates a sense and a separate object being sensed. Granted the experience of sensation (the conscious representation) may be particular and unique to the sense (the eyeball attached to a brain) and distinct from any object being sensed, but for strong solipsism to be coherent, we have to ignore the eyeball and the light that hits it, in which case why call this sensation? We are all just somehow fabricating everything we experience from some function of ourselves. Like a Berkleyan idealist.

    Weak solipsism it seems to me is the observation that our senses color and manipulate the separately existing world as it might be in itself, so we are cut off in a solipsistic experience, but this experience still provides some data that relies on the separately existing world exists (ie, there is a separate thing being sensed).

    There is no logical way around this that I have discovered (have to revisit Searle).
  • BC
    13.3k
    The underlying observation of Solipsism is that we only ever experience Sensory Data.Treatid

    You are underestimating sensory data.

    If you are relaxing at the beach, eyes closed, listening to the waves, day dreaming... and somebody dumps a bucket of ice water on you, that's a very compelling wash of sensory data. Or, if you wade out into the water and a shark bites your leg off, that's another compelling experience of sensory data. If you have enough pennies to dine at a really fine restaurant, you will have one course after another of splendid sensory data.

    Your senses have evolved over 600 million years of contact with the harsh sensory data of the world. In the case of ice water, shark bite, and fine food, it seems like you are in pretty direct contact with objective reality.

    Your mind isn't in direct contact, but your body is.
  • Treatid
    22
    Thank you for the feedback.

    @jkop, Thank you for the link to Searle. An intertesting read. While I agree with his qualms regarding perception I don't think his argument rises to the status of 'refutation'. It is a rebuttal but it isn't sufficiently definitive to be considered the undisputed status quo. And, as it happens, I think he is mistaken.

    {Searle presents the concept of perceiving a tree and hallucinating a tree. He argues that these two perceptions must be distinct - one 'sees' a tree the other 'not-sees' a tree. According to Searle, these two perceptions are distinct whereas previous philosophers conflated them into a single definition of 'see'.

    He admits that the perception for both real tree and hallucinatory tree are identical but then distinguishes between them. He declares one an hallucination and the other real without any basis upon which to make that distinction. He just 'magically' knows that one is a real perception and the other isn't.

    It is like arguing that two empty sets are distinct.

    {All empty sets in mathematics are indistinguishable. As such, by convention, there is just one empty set.}

    Searle is arguing that two indistinguishable perceptions are distinct becuase... he says so?}

    Clarification

    I think that objects do not exist.

    I think that Sensory Data does exist.

    I think that Objects and Sensory Data are distinct i.e. Object != Sensory Data.

    These statements can be taken as a statement of axioms or as a definition of terms. (More on definitions in just a moment).

    Solipsism is an interpretation based on observation and assumption.

    The observation (that we only experience Sensory Data) is not, itself, solipsism.

    Certainty

    "Cogito Ergo Sum" - René Descartes ("I think, therefore I am").

    This statement signifies that the only thing we can know with certainty is our own existence. The existence of everything else cannot be proven beyond all doubt.

    Solipsism takes this to an extreme and proposes that everything else is a figment of the imagination (or some equivalent).

    Existence

    You know that you exist. What does it mean to exist? What is the definition of existence? Why do you exist?

    If we haven't fully defined existence how can we be sure that we do, actually, exist?

    All standard philosophical fare that has been discussed on these forums with each person coming to their own conclusions that may or may not overlap significantly with anyone else's.

    At this point we can throw our hands up in disgust, declare ourselves nihilists, and give up on everything...

    Or we can work with what we have.

    What we have

    You exist.

    Your existence encompasses your entire existence. Everything you think, dream, feel and otherwise experience is part of your existence.

    Sensory Data exists.

    We haven't defined what Sensory Data is in any definitive sense. We experience something that we label Sensory Data. Just like we experience existence and label it Existence.

    We haven't shown that Sensory Data isn't just another name for Existence.

    This is what we have

    There are no exceptions. There are no loopholes. This is the nature of existence.

    LOOK! There! Did you see it?

    This is the nature of existence. We have a property of existence. We label parts of our existence without knowing for certain that these parts are distinct from the other components of our existence.

    If you were hoping for absolute, definitive, definitions then you are disapponted. But knowing what isn't is a piece of knowledge we can work with.

    Tie back

    Philosophers, mathematicans and physicists have been looking for definitive, absolute truths to build upon. Objective truths. An Objective Universe.

    Our (your) direct experience tells us that this isn't possible.

    When you tell me that I can't definitevely define Sensory Data you are right. I can't.

    This applies to every word and concept you can imagine.

    Look at the responses in this thread and see how much people are depending on particular definitions that they can't quite state in a definitive fashion... or simply saying such definitions are not possible.

    This is something you are already familiar with as philosophers. You already know that objective definitions are a hard problem.

    Don't fight this result. Lean into it. Accept it. Then work forward from there.

    Once you accept that there can be no definitive, objective, definitions philosophy (and mathematics and physics) become orders of magnitudes easier.

    "Once you stop trying to do the impossible, everything else (the possible) is trivial in comparison" - Misquoting Arthur Conan Doyle through Sherlock Holmes.

    Conclusion

    I have some sense of how this might come across but this is, at heart, a very simple argument:

    No-one has ever figured out how to definitevely define existence (or anything else). This is positive information about the nature of the universe we inhabit.

    Accept this insight. Work with it rather than against it.

    {granted - it may not immediately be clear how to use this information.

    It isn't as difficult as it might seem at first. We already don't have any definitive, absolute, definitions. Everything we have achieved thus far has been done in the absence of any single global definition.}
  • Kizzy
    99
    Consider this intel gathered from video attached below:

    Assuming a sensation always affects our emotional state. What is the difference between influence of emotional states created from exposure to direct sensations (reality) or illusion of it (indirect augmented sensations)?

    we assume a SENSATION ALWAYS affects our EMOTIONAL STATE....when we create a sensation *(see- "A simplified Model for Augmented Sensation: Defocus, Experience and Insperience") in our consciousness it has affect on emotional state, and now exposed to devices with screens and making experience without screens....we can see 2 sensations of 2 different structures.

    BOTH situations from pov of our conscious are seemingly like SENSATIONS, BUT CLEARLY HAVE DIFFERENT STRUCTURE WITH DIFF UNDERLYING PROCESS for creation of sensations involving ONLY external for high sense & external and internal sources for illusion of high sense....CONFUSION between experiences occur because the illusion of high dimensional sensation is created. Could be dangerous -- this implies different states of emotion, from knowing or not knowing about the process. (creation of augmented sensations- the illusion is the source of emotions from indirect external stimuli happening in the mind as the body is defocusing during this low dim sense created from just partial stimuli while direct external sources (outside world) full stimulation of senses occur (eyes ears nose taste touch) naturally as high dim senses are created through experiences faced in day to day life)

    A simplified Model for Augmented Sesations: Defocus, Experience, and Insperience
  • Treatid
    22
    @Kizzy,

    I've watched the video. I was tickled by the portmanteau of 'experience' and 'internal' to give 'insperience'.

    It isn't clear to me what problem you are seeing and what solution you are presenting. As such, I'm going to do some creative interpretation of what I think your point is while using the opportunity to expand on my own views.

    You seem to be covering similiar ground to the article by Searle that jkop linked earlier in the thread.

    I will assert that there is a common dogma in modern thinking that assumes the existence of a definite reality that exists separately from our subjective perception of it.

    You express this idea as a distinction between the emotional experience of real nature versus a different pseudo experience when seeing a movie or screen. Searle presents hallucinations as his version of psedo reality.

    You both believe/assume an objective reality that is distinct from our subjective experience of that reality and this informs your interpretations.

    While the belief in an objective reality distinct from our subjective perceptions is widespread - it is a belief without evidence.

    Integers

    Integers are whole numbers: 1, 2, 4, -6,...

    Except those examples aren't integers. '1' is a reference to an integer. A label for the concept of an integer. Integers themselves do not have physical form.

    Note: this isn't new. c.f. Platonic Ideals.

    So, Integers cannot be sensed as physical objects but they are still real. Aren't they?

    Integers are as real as God. "There is no way to measure them but they determine how things work."

    I am not saying anything about God. I am saying that the arguments used regarding the presence of God apply directly to Integers.

    Belief in Integers is equivalent to belief in God. You can believe in them if you wish but they are defined to be free from evidence.

    Note: Absolute faith without proof is a relativiely modern interpretation of Christian belief. It isn't, as I understand it, a requirement for belief. I am only interested in the related arguments for the existence of an invisible, undetecable, unmeasurable entity and applying those arguments to Integers.

    Definition of an Integer

    Definitions are hard. Mathematicians have invested a huge effort into defining basic concepts and the end result is that even their very best definitions contain a disturbing amount of handwaving

    Fortunately we don't have to get to complex.

    The Laws of Thought are: The Law of Identity, The Law of the Excluded Middle and The Law of non-Contradiction.

    These are the explicit axioms (assumptions) of Axiomatic Mathematics which is the branch of mathematics that contains all the Proofs (including Formal Logic).

    The integer '1' obeys the first Law of Thought (Identity). 1 is always itself. It doesn't change. 1=1.

    All well and fine.

    Now... do something constructive with that integer.

    You could.... change it into... itself.

    And that is it. That is everything you can do with the integer by itself. You can imagine it sitting there doing nothing.

    The problem with static, unchanging objects is that they are static. They don't do anything.

    Change

    Previously we considered that your existence is self-evident; and includes your entire existence.

    An aspect of your existence is your awareness of the existence of change. Whatever the exact nature of change, you are as certain of its existence as you are your own existence.

    Mathematics is the discipline of describing a changing universe based on the assumption that things don't change.

    Note: This applies to Axiomatic Mathematics which is not all mathematics.

    An electron

    Electrons are just like Integers.

    No-one has ever seen an electron. It is impossible to measure the properties of an electron.

    Again, this is not a new revelation. Electrons are a hypothetical particle invented to explain the observations we make. This is what a theory is.

    Except that electrons don't change. The same axioms for Axiomatic Mathematics are part of the mathematics of Quantum Mechanics (The Standard Model of Physics).

    Note: Quantum Mechanics is an incredibly successful theory that predicts observations to incredible levels of accuracy and precision. However, there is no way to start with the assumption of Identity (non-changing) and arrive at a changing universe.

    This is a straightforward contradiction which, in mathematics, is devestating for the theory.


    Quantum Mechanics does work - but it cannot possibly work according the the stated mechanisms.

    Sensory Data

    Sensory Data (whatever it actually is) changes.

    Sensory Data isn't a window into the Real Objective Universe. Sensory Data is that universe.

    Our experiences aren't a translation of an objective universe into subjective experience. Our experiences are the universe. Not in a solipsistic "we make the universe sense".

    In a - What We Experience Is What Is There (WWEIWIT) way.

    We don't need to infer reality from our Sensory Experience. Our Sensory Experience is our direct experience of reality.

    Objects in Mathematics are defined as unchanging.

    Sensory Data changes.

    You know this. You can see this.

    This isn't complicated. Do you experience change?

    If yes: you do not live in an objective universe.

    The idea of a fixed, objective universe is attractive. It makes arguments easier when the target isn't moving around all over the place. But it isn't true. It is obviously not true.

    You can try describing unchanging objects until you are blue in the face - but it will get you precisely nowhere in understanding a changing universe.
  • Thales
    18
    "Cogito Ergo Sum" - René Descartes ("I think, therefore I am").

    This statement signifies that the only thing we can know with certainty is our own existence. The existence of everything else cannot be proven beyond all doubt.
    Treatid

    Actually, whereas Descartes may have proven “thinking” exists, his leap to proving his own existence is less certain. He argued:

    “I will doubt everything I see, hear, smell, touch, taste and think until I come to a place where doubt is not possible. As it turns out, the only thing I can’t doubt in this list is ‘thinking’ because ‘doubt’ is, itself, a form of thinking.”

    But just because it was proven that “thinking” exists, it doesn’t necessarily follow that he (Descartes) exists. Thinking does not (logically) imply personal identity.
  • Philosophim
    2.4k
    Philosophers, mathematicans and physicists have been looking for definitive, absolute truths to build upon. Objective truths. An Objective Universe.Treatid

    This is something you are already familiar with as philosophers. You already know that objective definitions are a hard problem.

    Don't fight this result. Lean into it. Accept it. Then work forward from there.
    Treatid

    Not at all. Being intellectually lazy and giving up to pat ourselves on the back is what the general populace does. Don't ever fall for that intellectual trap. Your mind, like your muscles, wants to be lazy, sit on a couch and get fat. Don't let that happen. If you're interested in epistemology, I have at least one objective definition here. There's a good summary in the following post if that helps.
    https://thephilosophyforum.com/discussion/14044/knowledge-and-induction-within-your-self-context/p1
  • Treatid
    22
    Actually, whereas Descartes may have proven “thinking” exists, his leap to proving his own existence is less certain. He argued:Thales

    I don't think 'proof' is relevent here. Or at least not as the result of a chain of Logical Deduction.

    Your existence is evident to you. Not as a matter of argument but as a matter of experience.

    If we get into the weeds - we don't know what 'thinking', 'existing', 'experience' or 'self' mean in a definite manner.

    Descartes' statement is more along the lines of: "There is definitely something/I have a sense of self".

    The 'Ergo' (therefore) part of Descartes' statement seems to me to be a mistake. He doesn't exist because he thinks. His thoughts are an aspect of his existence.

    This may feel a little nihilistic if we interpret this as "we know something but we can't define what that something is".

    I think people expect to be able to define things in definitive terms and give up when it turns out that isn't possible.

    My argument is that this is a piece of knowledge that we can work with. It isn't possible to define anything in absolute terms. What systems can function in the absence of concrete definitions? It is evident we inhabit a system that works without having absolute, fixed definitions.


    People are good at Categorising things. It is a powerful tool and has almost certainly been instrumental in our success as a species.

    I like the style/rigour of what you are doing. But I think what you are trying to do is impossible.

    No matter where you start - it is impossible to create definite, unambiguous definitions.

    Yes, it would make communication clearer and faster if we had rigorous definitions that everyone understood and agreed with. That isn't reality. People have been trying to create a solid, unimpeachable foundation to build on since forever. They haven't succeeded because it is an impossible task.

    For example, I think you cannot justify the distinction you make between thought and experience.

    Thought and experience are aspects of a single whole. You can't have thought without experience and vice versa.

    Wordlviews

    1. There are objects with properties that give rise to relationships between objects.

    2. There are relationships.

    Both these worldviews agree on the existence of relationships. Adherents of the first worldview attempt to explain observed relationships through the properties of objects.

    The second worldview describes the nature of relationships by describing relationships.

    Relationships are a fundamentally distinct concept from objects.

    An object is a static singleton. An object has properties. These properties are also static. These static properties give rise to dynamic relationships... somehow?

    A relationship, in contrast, changes. A relationship connects. A relationship presents difference.

    It is not possible to build a relationship using only objects.

    Fundamental Unit

    The fundamental unit of the universe is a relationship.

    The universe is a network of relationships that changes.

    Why? Because we can see that is what it is.

    The universe changes, so it must be composed of stuff that can change. The universe is connected so it must be composed of things that connect. The universe is diverse so it must be composed of differences.

    Objects do not have these properties. The universe is not composed of objects.

    We label the things Relationships.

    Language

    Language builds networks of relationships.

    Why? Because language is part of the universe. The basic ingredients of the universe are the basic ingredients of everything in the universe.

    It is these networks of relationships that convey meaning.

    Individual words are placeholders for other networks of relationships. We can connect existing networks of relationships together to build new networks.

    Any given statement is a network of relationships.

    A network of relationships, by itself, is just a shape. Words on the page are just shapes. Shapes don't intend anything.

    When you read you incorporate the shape of the language into your own shape (you, also, are a network of relationships). The way you incorporate new shapes into yourself depends on your existing state and the way it interacts with the new shape.

    Corollory

    Static objects cannot describe a dynamic universe.

    At the same time, Relationships cannot describe static objects.

    Descriptions work by describing unknown objects using altered descriptions of known objects.

    A unicorn is a horse with a horn. You know what a horse is, you know what a horn is. By combing the two we convey what a unicorn would look like.

    If you were to experience something without any precedent you wouldn't be able to convey your experience.

    "It was like nothing you have ever seen before."

    Challenge Time

    If you can describe a static object you will have shown that I'm wrong and that I don't know what I'm talking about.

    The two main arguments I'm going to fall back on will be:

    A. You haven't actually described anything. "Objects are not relationships" is not a description of an Object.

    B. What you have actually described is relationships. My default position is that if you manage to describe something it must have actually been a (set of) relationsips in the first place.


    This is, of course, a blatant attempt to get you to engage with the ideas of what language is capable of and what it isn't capable of.

    Can you describe something that has no similarities to any of your previous experiences?
  • Joshs
    5.4k


    The fundamental unit of the universe is a relationship.

    The universe is a network of relationships that changes.

    Why? Because we can see that is what it is.

    The universe changes, so it must be composed of stuff that can change. The universe is connected so it must be composed of things that connect. The universe is diverse so it must be composed of differences.

    Objects do not have these properties. The universe is not composed of objects.

    We label the things Relationships.
    Treatid

    Kind of like this from physicist Karen Barad?

    “In an agential realist account, matter does not refer to a fixed substance; rather, matter is substance in its intra-active becoming—not a thing but a doing, a congealing of agency. Matter is a stabilizing and destabilizing process of iterative intra-activity. Phenomena—the smallest material units (relational “atoms”)—come to matter through this process of ongoing intra-activity. “Matter” does not refer to an inherent, fixed property of abstract, independently existing objects; rather, “matter” refers to phenomena in their ongoing materialization. On my agential realist elaboration, phenomena do not merely mark the epistemological inseparability of “observer” and “observed”; rather, phenomena are the ontological inseparability of agentially intra-acting “components.” That is, phenomena are ontologically primitive relations—relations without preexisting relata. The notion of intraaction (in contrast to the usual “interaction,” which presumes the prior existence of independent entities/relata) represents a profound conceptual shift. It is through specific agential intra-actions that the boundaries and properties of the “components” of phenomena become determinate and that particular embodied concepts become meaningful.”

    “In my agential realist account, scientific practices do not reveal what is already there; rather, what is ‘‘disclosed’’ is the effect of the intra-active engagements of our participation with/in and as part of the world’s differential becoming. Which is not to say that humans are the condition of possibility for the existence of phenomena. Phenomena do not require cognizing minds for their existence; on the contrary, ‘‘minds’’ are themselves material phenomena that emerge through specific intra-actions. Phenomena are real material beings. What is made manifest through technoscientific practices is an expression of the objective existence of particularmaterial phenomena. This is, after all, a realist conception of scientific practices. But unlike in traditional conceptions of realism, ‘‘objectivity’’ is not preexistence (in the ontological sense) or the preexistent made manifest to the cognitive mind (in the epistemological sense).

    Or this from Deleuze and Heidegger?

    In accordance with Heidegger's ontological intuition, difference must be articulation and connection in itself; it must relate different to different without any mediation whatsoever by the identical, the similar, the analogous or the opposed. There must be a differenciation of difference, an in-itself which is like a differenciator, by virtue of which the different is gathered all at once rather than represented on condition of a prior resemblance, identity, analogy or opposition.

    Or Deleuze’s summary of Nietzsche’s Eternal Return?

    When the identity of things dissolves, being be­gins to revolve around the different. That which is or returns has no prior constituted identity: things are reduced to the difference which fragments them, and to all the differences which are implicated in it and through
    which they pass
  • Philosophim
    2.4k
    I like the style/rigour of what you are doing. But I think what you are trying to do is impossible.Treatid

    I appreciate it, but we're on the philosophy boards. We can claim things are possible or impossible, but its all about proving it. Can you prove its impossible? What you're doing otherwise is taking the weak position of trying to have everyone else prove something which you haven't clearly spelled out yourself. You're committing the very flaw you're accusing others of.

    Yes, it would make communication clearer and faster if we had rigorous definitions that everyone understood and agreed with. That isn't reality.Treatid

    So if you read the paper, you'll find that I both form foundations and standards upon which we can have definitive knowledge. Yet I also note there are plenty of times (in fact, the majority) in which much of our claims, language, etc, are based on induction. This is necessary for efficient communication, as creating a solidly known definition takes time and effort that is generally not required in day to day conversation. That does not mean it is impossible to create objective knowledge or terms.

    So yes, most of our communication is filled with inductive premises, but that's a far cry from stating it is impossible to create or think on something objective. Again, you need to prove this yourself, not take the weak stance of making everyone else do the work to counter a claim you haven't fully tested yourself.

    For example, I think you cannot justify the distinction you make between thought and experience.

    Thought and experience are aspects of a single whole. You can't have thought without experience and vice versa.
    Treatid

    Then you misunderstood the definitions. Thoughts are an experience. I never say otherwise. "All tigers are cats, but not all cats are tigers." for example.

    If we get into the weeds - we don't know what 'thinking', 'existing', 'experience' or 'self' mean in a definite manner.Treatid

    Yes, you do. You do within your own experience. Words are tools we use to categorize discrete experiences and concepts. They can be loosely defined, or very tightly defined. That's your call. If you wish to define something to the point it can be objective, you can, it just takes a lot of effort. To communicate with others, there must be a certain level of rigid definition to the term that both of you share, or else communication would be impossible. If I say 'dog' but my personal definition of dog is 'monkey', then we're at a loss with each other.

    But if I'm a botanist who studies tree species, I'm going to have very rigorous definitions and standards that I share among my co-workers. These are objective terms used to identify plant species. Without this, science would be worthless.

    While you get that definitions and communication are contextual, the contextual does not mean we can't have objective definitions. The contextual decides how objective our definitions are required or expected to be in that particular context. Just because you choose to remain in a context that you decide not to use objective definitions, does not mean its impossible for there to be a context in which there are tight objective definitions and conclusions.

    If you can describe a static object you will have shown that I'm wrong and that I don't know what I'm talking about.Treatid

    Certainly, a 1X1X1 inch cube of solid iron with a density that weighs 10 grams.

    A. You haven't actually described anything. "Objects are not relationships" is not a description of an Object.Treatid

    Its iron, its in the shape of a cube with 1X1X1 dimensions, and it weighs 10 grams. This can be tested and confirmed objectively.

    B. What you have actually described is relationships. My default position is that if you manage to describe something it must have actually been a (set of) relationships in the first place.Treatid

    The universe changes, so it must be composed of stuff that can change. The universe is connected so it must be composed of things that connect. The universe is diverse so it must be composed of differences.

    Objects do not have these properties. The universe is not composed of objects.

    We label the things Relationships.
    Treatid

    I'm going to fire back here and note that your definition of "Relationship" needs to be tighter. If you're going to dismiss normal use of commonly accepted words and introduce your own meaning, you need to be very clear for it to be accepted. Objects do not have relationships like people do. Objects are described and known by experience and properties. If this simple language does not work, you need to detail why.
  • Treatid
    22
    Kind of like this from physicist Karen Barad?Joshs

    It certainly seems like it at first glance. I will look into her in more detail.

    Thank you very much for this reference.

    I appreciate it, but we're on the philosophy boards.Philosophim

    I'm about to come in hot. I can do this because you are making clear statements of position that I can engage with.

    Thank you for that.

    We can claim things are possible or impossible, but its all about proving it.Philosophim

    No. It isn't.

    Remember solipsism?

    It is impossible to prove anything beyond all doubt (except, perhaps, your own existence is self-evident to you).

    To the best of my knowledge there is no accepted counter argument.

    I'm sure many people think solipsism is silly, bordering on nihilism. That doesn't mean the underlying observations are wrong.

    There are no (logical or mathematical) proofs.

    One may not like this observation. This observation doesn't care.

    It is the nature of the universe that you cannot have definite proofs (as defined by formal logic and axiomatic mathematics).

    If you genuinely want to understand the nature of the universe then, sooner or later, you are going to have to come to grip with the fact that this is the nature of the universe.

    Rant

    I want to rant longer and harder because this is such an important point.

    I suspect that solipsism is a deliberately obtuse interpretation of the observations in order to make it appear less relevant. But it isn't wrong.

    Solispsism destroys Axiomatic Mathematics. Even in the absence of an alternative, solipsism is a clear statement that we must go back to the drawing board.

    It isn't even subtle.

    The mental gymnastics required to adhere to Axiomatic Mathematics in the face of (the observations leading to) solipsism is truly world leading. And this is from (rational) mathmaticians.

    This (Proofs/Axiomatic Mathematics) goes beyond "Absolute belief without proof". It is "asbolute belief despite overwhelming evidence to the contrary).

    Belief in the existence of proofs is religious in nature.

    I've no doubt there was a time when it seemed to be a rational approach to knowledge. Then Descartes happened.

    {Actually, the principles of solipsism were recorded over two thousand years ago. Descartes was more of a brush up than a genesis of the ideas. We've had two millenia to get our house in order. It is, perhaps, time we started facing upo to the truth no matter how uncomfortable that may be.

    (It really isn't that uncomfortable. Once you stop trying to do impossible things and go with the flow (possible things); it is astonishing how quickly all the pieces fit together).}


    In conclusion

    Do you have a specific reason why we should disregard solipsism and the observations that lead to it?
  • Joshs
    5.4k
    Do you have a specific reason why we should disregard solipsism and the observations that lead to it?Treatid

    I can think of reasons to disregard the following definition of solipsism from Enclyclopedia Brittanica:

    In philosophy, solipsism is an extreme form of subjective idealism that denies that the human mind has any valid ground for believing in the existence of anything but itself. The British idealist F.H. Bradley, in Appearance and Reality (1893), characterized the solipsistic view as follows

    “I cannot transcend experience, and experience must be my experience. From this it follows that nothing beyond my self exists; for what is experience is its [the self’s] states.”

    My critique centers on the Idealist conception of self expressed in the definition. If we must remain skeptical about the existence of everything but the sense data we experience, what kinds of presuppositions are invoked in talk of an ‘I’, a self that has these experiences, and where do these presuppositions come from? Do they come from experience or do they force a certain account onto experience, a certain interpretation of sense data wherein a self remains absolutely fixed as the subject of experience, reflecting back on itself as self-identical over time?

    If all sense data are fundamentally in a state of changeable, interrelational becoming, then isnt this also true of the self, subject, ego, I? Does any notion of self make any sense outside of its inextricable relations with others? Plenty of philosophical positions deconstruct this notion of the absolute unchanging self in favor of a self which is constructed though social interaction. For them the self is a continually changing product of these interactions rather than an unchanging substance. You can also check out this ongoing thread:
    https://thephilosophyforum.com/discussion/15267/concept-of-no-self-in-buddhism
  • Philosophim
    2.4k
    I'm about to come in hot. I can do this because you are making clear statements of position that I can engage with.

    Thank you for that.
    Treatid

    No problem! This is the place where you can. Please enjoy thinking and challenging to your hearts content.

    It is impossible to prove anything beyond all doubt (except, perhaps, your own existence is self-evident to you).Treatid

    I doubt that. And if doubt had any weight on its own, your statement would crumble. Doubt alone means nothing. I can doubt anything I want. What matters is "Reasonable doubt". If I say, "I doubt that unicorn's aren't real," someone can ask me, "What's your evidence for such a doubt?" If I say, "Because I want there to be unicorns," my doubt is just a feeling and can be dismissed.

    Doubt is never the standard by which objectivity is determined. After all, people doubt that the world is a sphere. Doubts which can be tested, are falsifiable, and have good reasoning behind them can be considered as viable challenges to established claims. Anything else can easily be discarded.

    There are no (logical or mathematical) proofs.Treatid

    Can you prove this? You're running into a classic conundrum of, "There is no reality/certainty etc." Your own statements fall into your own accusation. In which case, why should your statements hold any weight?

    Look at it this way.

    Option 1: There is not objectivity, so anything goes.
    Option 2: There is objectivity, so not everything goes.

    If you pick option 1 and I pick option 2, I don't have to accept your premises, while you have to accept that its fine for me to hold option 2. If you insist that I'm wrong, then you've countered your own argument. You're claiming, "Option 1 is objectively true, while option 2 is objectively false." Or better/relative comparative value that is more than an opinion. Option 1 is rationally worthless.

    So while I could demonstrate a logical or mathematical proof, I'm instead putting the onus on you. You do the work of proving why option 1 doesn't collapse into pointlessness. And by doing so, option 1 crumbles. If you don't prove it, then I get to ignore it. If you do prove it, then you counter your own argument.

    It is the nature of the universe that you cannot have definite proofs (as defined by formal logic and axiomatic mathematics).Treatid

    Again, feel free to prove this. Once again, doing so will defeat the statement.

    Solispsism destroys Axiomatic Mathematics.Treatid

    Once again, the onus is on you to prove this statement.

    Do you have a specific reason why we should disregard solipsism and the observations that lead to it?Treatid

    I have plenty, but first you must demonstrate why there is any reason we should accept solipsism and the observations that lead to it. Why should I, a person who isn't you, accept that you are able to prove that the only thing I can know exists is me?
  • Treatid
    22
    My argument centers on the Idealist conception of self expressed in the definition.Joshs

    I... agree with what you are saying.

    I'm confused as to why you think this is an argument against solipsism or its' underlying observations.

    You point out that the definition of 'I' or 'self' is unclear. I agree with this.

    I think you are then making an (unstated) assumption that if we cannot define the strict meaning of words then arguments involving those words are meaningless and we shall all just give up.

    Why can't we strictly define words?

    Imagine a closed system that you are part of. You are not outside the system looking in. You are inside the system. You are part of the closed system.

    You wish to talk about this system you inhabit.

    Anything you say is within the system. If you point at something, it is inside the system.

    If you try to describe the system as a whole, your description is inside the thing it is trying to describe.

    You would be using the universe to describe the universe.

    It is like trying to describe a sheep using only references to that sheep: "A sheep's head looks like... the sheep's head." "The Sheep's wool is soft like the sheep's wool and covers the body in the way that the sheep's wool covers its' body."

    You can get more creative: "The sheep's head is smaller than its body." "The tail is at the opposite end to the head."

    This is, of course, where we are. We are inside the universe.

    We can take one piece of the universe (a one metre ruler) and compare it to the circumference of the Earth through the poles. We can say this circumference is a little over 40,000km.

    This works.

    What is distance? What creates distance? Given nothing (a formless void) how would you create distance? How did distance come into being?

    An electron

    An electron has properties we describe as wavelike. What is a wave? A wave is something we observe elsewhere in the universe.

    An electron has properties that are like other parts of the universe.

    Okay. One piece of universe has similarities to other pieces of the universe.

    This isn't nothing. We can enumerate those similarities and differences. We can measure and compare similarities.

    However, sooner or later we need to admit that we have described a property of electrons using the result of those electrons. An ocean wave is composed of (among other things) electrons. We then describe properties of the electron using that macroscopic idea of waves.

    We are describing electrons using electrons.

    Definitions

    The above is true of all definitions. We can compare (measure) different distances. But if we try to define what distance is we end up saying that distance is like this other thing that we can't define.

    Assumption: definitions work by comparing and contrasting

    It is extremely obvious to me that all descriptions describe one thing in relation to other things. "A zebra is like a horse with stripes."

    If this doesn't seem obvious to you I would love to get some insight into how you think information is conveyed.

    Next

    There are no (logical or mathematical) proofs. — Treatid


    Can you prove this?
    Philosophim

    A: "There are no proofs."

    B: "You must prove to me that there are no proofs for me to accept your statement."

    Proof is the de facto standard of modern scientific argument. It is the accepted mechanism of ensuring that an argument is rigorous and actually justifies the claims made.

    Pushing for that rigour is justified.

    Cutting yourself off from any mechanism that would question 'proof' and 'logic' is anti-rational.

    You have defined a position that is unassailable. "If there are no proofs then it is impossible to prove there are no proofs. Therefore, obviously, the proposition that there are no proofs is wrong Q.E.D."

    A proof of the non-existence of proofs

    A contradiction in Logic disproves the axioms.

    Logic (and Axiomatic Mathematics) start with a set of premises (axioms) which define an initial condition and a set of rules for reaching new statements within that system. If a statement within the system is both true and false at the same time, then there is some fault with the initial axioms. The Premise is mistaken.

    The universe is a system. If a contradiction were to appear inside the universe then, logically, the universe must disappear in a puff of logic.

    According to Axiomatic Mathematics, there are many inconsistent systems. These systems exist within the universe. Why hasn't the universe poofed out of existence.

    Axiomatic Mathematics part 2: Separation

    Axiomatic Mathematics (and formal logic) are instantly dead if a single contradiction invalidates the entire system.

    So each Axiomatic System must be independent of every other Axiomatic System. An individual system may be invalidated but this has no impact on all the other independent systems.

    This works great.

    It is a Lie

    The separation between Axiomatic (or Logical) systems doesn't exist.

    There is no measurable quality that demonstrates the distinction between two systems.

    A mathematician has to tell you that two sets of statements are distinct. There is nothing you can see, feel, touch or hear that will tell where to draw the boundary between axiomatic systems.

    Axiomatic mathematics must have distinctions between systems in order to exist. But the boundaries can't be seen. There is nothing to measure. They are the poster child of belief without evidence.

    If everything is connected then Logic, Axiomatic Mathematics and the whole universe are inconsistent. By the rules of Axiomatic Mathematics inconsistent systems have no information content.

    Axiomatic Mathematics needs a distinction between systems to exist.

    Good luck trying to demonstrate one of these distinctions.

    Where we are going we don't need proof

    Logic is a theory of arguments. It (tries to) describe how a form of communication works.

    Before communication we have experience.

    Descartes didn't argue your self awareness into existence.

    Before you can argue about what it means to exist, what 'self' is or all the rest of it; you first need to exist (whatever that means).

    As far as certainty goes - your existence is the pièce de résistance. There is nothing better. It is all downhill from here.

    Caveat

    Your existence encompasses the whole of your existence. All your experiences are part and parcel of your existence. You are as certain of your direct experiences as you are of anything else.

    Logic never persuaded you that you feel pain and pleasure. You feel pain and pleasure because... you do.

    We can (indeed, must) use our personal experiences as the solid foundation upon which to build... everything.

    Solipsism says we cannot know anything with certainty except the self.

    This isn't wrong - but the self includes everything you ever experience. When you stub your toe on a table; that experience is certain. Definite.

    If we only talk about your experiences; we are limited to everything you can possibly experience.

    You have only ever been able to talk about your experiences.

    "The only certainty is your own existence." isn't a statement of limitation. Your existence is EVERYTHING of significance.

    P.S. The universe is consistent

    You can't cause bits of the universe to evaporate by making the wrong symbols.

    There are no statements that have to be wrapped in a little knot of pearl in order to prevent the death of the universe.

    A physical sentence isn't wrong. It isn't right either. These words don't have any meaning. They are just shapes in the universe.

    When you read these words you decide on their meaning and their significance.

    If you decide that a sentence is wrong - fine. It is still just a bit of universe shaped in a particular way.

    The universe doesn't think the liar's paradox is a paradox. It is just squiggles on (virtual) paper.

    Not all the squiggles on paper make sense to the people reading them. This isn't a squiggle problem. This is a people problem.

    You, personally, decide how you will respond to what you read.

    You have never been persuaded by Logical Argument. You have found squiggles that made sense according to your personal experience.

    That is the final arbiter of your understanding. Your experience determines what you find plausible and implausible.

    Right and wrong (truth and false) are entirely subjective opinions determined by each individual.

    The symbols on the page are just symbols on the page. A symbol isn't true or inconsistent.

    Your interpretation of a symbol is not the symbol.

    P.P.S.

    Once we get past the idea that symbols have any kind of inherent meaning; we can start to consider how similarities of experience between people enables the use of those 'meaningless' symbols to communicate meaning to other people.

    Which is to say - of course language works. Of course we are able to communicate.

    That doesn't mean the mechanism bears any similarity to the fairy tale we, as a society, have been telling ourselves for far too long.
  • Joshs
    5.4k


    I'm confused as to why you think this is an argument against solipsism or its' underlying observations.

    You point out that the definition of 'I' or 'self' is unclear. I agree with this.

    I think you are then making an (unstated) assumption that if we cannot define the strict meaning of words then arguments involving those words are meaningless and we shall all just give up.
    Treatid

    I was only introducing a commonly accepted definition of solipsism, which isn’t unclear at all, and wondering if it corresponds to your use of the word. And if it doesn’t, how does your use differ? You keep on referring to me my, I. Would you be amenable to getting rid of these terms and instead just describing a constantly changing center of activity that we mistakenly refer to as a ‘self’? What other things do you believe can be definitively said about this center of experiencing without having to dip into mathematics and logical axioms?

    Can we say this center has memory , consciousness of a past, present and future? If we throw out the language of propositional logic and math , aren’t we still able to keep a range of neuro-psychological descriptions of human experiencing? Is your purpose in this thread simply to critique the assumed pre-eminent role of math and logic in the ascertaining of truth ( in which case you have a lot of company, not only in philosophy but in the social sciences)? Or is your aim also to critique what you understand to be the cutting edge of ideas in philosophy and the sciences ( in which case you run the risk of reinventing the wheel)?
  • Philosophim
    2.4k
    If a contradiction were to appear inside the universe then, logically, the universe must disappear in a puff of logic.

    According to Axiomatic Mathematics, there are many inconsistent systems. These systems exist within the universe. Why hasn't the universe poofed out of existence.
    Treatid

    True. Do you have a specific example of when something both existed and did not exist at the same time in the universe? And I don't mean metaphysically, but in reality.

    Second, you'll need to point out a contradiction in math, and prove that it necessarily represents the universe accurately. A claim of 'inconsistent systems' is not enough.

    The separation between Axiomatic (or Logical) systems doesn't exist.Treatid

    Correct. Logical systems are built upon smaller proven parts.

    Axiomatic mathematics must have distinctions between systems in order to exist. But the boundaries can't be seen. There is nothing to measure. They are the poster child of belief without evidence.Treatid

    I don't understand what you're stating here. Could you give an example?

    If everything is connected then Logic, Axiomatic Mathematics and the whole universe are inconsistent. By the rules of Axiomatic Mathematics inconsistent systems have no information content.

    Axiomatic Mathematics needs a distinction between systems to exist.
    Treatid

    How is everything inconsistent? I also don't understand your claim that axiomatic mathematics needs a distinction between its 'systems'. (What's a system?)

    Logic is a theory of arguments. It (tries to) describe how a form of communication works.Treatid

    No, logic is a combination of proofs that have not been reasonably countered. Like I mentioned earlier, you cannot have an object 'A' both exist and not exist at the same time. A != A is proven to be wrong.

    Your existence encompasses the whole of your existence. All your experiences are part and parcel of your existence. You are as certain of your direct experiences as you are of anything else.Treatid

    No disagreement here.

    Logic never persuaded you that you feel pain and pleasure. You feel pain and pleasure because... you do.Treatid

    No, but logic can give something more reasonable to stand on then a feeling of certainty.

    We can (indeed, must) use our personal experiences as the solid foundation upon which to build... everything.Treatid

    We only need personal experiences if we only trust ourselves. If we trust proven systems, then no.

    Solipsism says we cannot know anything with certainty except the self.

    This isn't wrong - but the self includes everything you ever experience. When you stub your toe on a table; that experience is certain. Definite.
    Treatid

    Yes, the experience of existing is certain as you experience it. But whether that experience correctly interprets how you exist in relation to others is not. Descartes evil demon covers this. Or the brain in a vat argument. Just because you believe strongly that you've stubbed your toe is not a reason alone to conclude you stubbed your toe. That's just an emotional conviction.

    A physical sentence isn't wrong. It isn't right either. These words don't have any meaning. They are just shapes in the universe.Treatid

    No, they have meaning by the concepts they represent. And those concepts can be right or wrong depending on if they are in accordance with reality, or contradict reality.

    You, personally, decide how you will respond to what you read.Treatid

    Sure. But this doesn't prove that my belief of certainty matches reality. You're equating emotions and beliefs with logic. They are not the same.

    Right and wrong (truth and false) are entirely subjective opinions determined by each individual.Treatid

    It depends on the context. If I want to make up names for things I can. If I want to say that a rock hurtling through the air is about to hit me in the head, I am wrong.

    I highly encourage you to read the work I linked you in the last post. You're the kind of person who can think along those lines, and I think it would be very valuable to help you sort out the thoughts your currently going through.
  • Treatid
    22
    Is your purpose in this thread simply to critique the assumed pre-eminent role of math and logic in the ascertaining of truth ( in which case you have a lot of company, not only in philosophy but in the social sciences)? Or is your aim also to critique what you understand to be the cutting edge of ideas in philosophy and the sciences ( in which case you run the risk of reinventing the wheel)?Joshs

    Part 1

    Bear with me - this may not seem like I'm answering your question but I have a plan.

    The universe is exactly what we perceive. Alternatively, existence is what we experience.

    There are a bunch of (or one large) mistaken assumptions that are blocking our collective ability to see clearly.

    For example, it is impossible to describe non-Sensory-Data.

    All the time and effort expended on trying to do impossible things is wasted. Worse, our expectations for what the answers should look like are impossible. So, even when a correct answer is staring us in the face we reject it because it doesn't conform to our (impossible) prejudices.

    On the flip side, our direct experience is informed by being part of a functioning universe.

    We already know that our personal experience is subjective. The idea of subjectivity isn't new or surprising.

    Trying to understand the nature of subjectivity from an objective viewpoint is futile.

    The only way to accurately describe the universe is from a subjective perspective. Trying to force subjective experience into an objective framework just causes confusion.

    You keep on referring to me my, I. Would you be amenable to getting rid of these terms and instead just describing a constantly changing center of activity that we mistakenly refer to as a ‘self’?Joshs

    There is one existence and everything is an aspect of that existence.

    This isn't some New-Age metaphor about how we should all live in harmony.

    Every part of the universe is connected. There is no clear delineation where one part ends and another part begins.

    Again, this isn't an original idea. The difference is that we aren't just tossing off a neat idea and moving on. The connectedness of the universe is an essential, fundamental trait. The fundamental (smallest) component of the universe must include the concept of connectedness. Connectedness has to be baked into the very fabric of the universe, it isn't an emergent behaviour that can arise from not-connected things.

    A practical upshot of this is that our understanding of any single concept is determined by the sum total of all our other concepts.

    For All A and All B
    {
    A is the difference between A and B.
    B is the difference between B and A.
    }

    Note that things that are connected must also be different. Connections don't just connect, they also differentiate.

    Following from this we get a sense of what 'self' is that isn't that far from your intuition, I think.

    Agreement

    I'm fairly confident that your subjective experience of 'self' is very similar to my experience of 'self'.

    If we were to discuss matters of existence and identity in purely subjective terms we could rapidly reach consensus and seek out new avenues of exploration.

    The trouble is that there is an expectation that valid conclusions must take the shape of 'objective statements' in a 'logical structure'. Subjective experience cannot be described in an objective framework.

    It is an impossible requirement.

    Language is, of course, already subjective.

    This leads to situations where people successfully discuss ideas (using languages that are already subjective) and then get stuck when trying to express those ideas in an objective/logical framework. They can feel that they are communicating and then get frustrated when they can't fit the round peg of subjective experiences into the square hole of objective definitions.

    I was only introducing a commonly accepted definition of solipsism, which isn’t unclear at all, and wondering if it corresponds to your use of the word. And if it doesn’t, how does your use differ?Joshs

    That is a fine definition of solipsism.

    Bear in mind that I've stated that the self encompasses your entire experience.

    This is, among other things, a question of knowledge. What do we know? What can we know?

    Solipsism represents the line in the sand. You can know yourself and the direct experiences that are part of yourself. You cannot know anything else.

    Attempting to know 'anything else' is a waste of time.

    So, what about the existence of other people?

    You have experiences that you associate with the concept of other people. Those experiences exist. There is no question about the existence of those experiences.

    The non-sequitur arises when you question whether those people exist beyond your perception of them.

    The closest I think I can parse this is: "Do the people that I experience exist outside the universe?"

    You don't have to prove that people exist outside of your experience. That is irrelevant. All the things that you cannot and do not experience have exactly zero impact on you.

    Note: I know you weren't arguing for or against solipsism. I am taking every opportunity to underline the point that subjective experiences are real. Subjective experiences are the bedrock of your existence. non-subjective-experiences are irrelevant. You will never experience objective experiences.

    Trying to fit your subjective knowledge into an objective framework is a waste of effort and time.

    Is your purpose in this thread simply to critique the assumed pre-eminent role of math and logic in the ascertaining of truth ( in which case you have a lot of company, not only in philosophy but in the social sciences)? Or is your aim also to critique what you understand to be the cutting edge of ideas in philosophy and the sciences ( in which case you run the risk of reinventing the wheel)?Joshs

    Part 2

    More ambitious.

    The critique is somewhat incidental albeit a necessary step.

    We have two world-views that are fundamentally incompatible with each other. There is no incremental (step-wise) path from one world-view to the other.

    Given objectivism it is impossible to fully grasp relativism (and vice versa).

    We've seen this before between Newtonian Mechanics and General Relativity. General Relativity is not an evolution of Newtonian Mechanics. They are incompatible systems.

    General Relativity is built up from first principles using only the observation that "the speed of light in vacuum is the same for all observers".

    By far the largest obstacle to understanding General Relativity is trying to interpret it from the perspective of (objective) Newtonian Mechanics.

    We are continuing the work of General Relativity except that our single observation is solipsism.

    As such, I have two purposes:

    1. Use the principles of solipsism to illustrate a relativistic system of thought.
    2. Undo the mistaken assumptions of objectivism which obscure the relativistic nature of the universe.

    Of these two tasks, the second is the most difficult by orders of magnitude.

    The universe is relativistic. Describing the universe (including things like existence, meaning and thought) in relativistic terms is trivial. For comparison, describing the universe in objective terms is impossible.

    Describing a relativistic system in relativistic terms is a joy. Everything fits together and just works.

    But if you are still holding onto objective assumptions nothing makes any sense. Proof is an artifact of objective assumptions. It isn't meaningful in a relativistic system.

    In a relativistic system, what you observe is the relativistic system. Your existence isn't proven by a chain of logical statements. Your existence is your observation of your existence (or more simply - you are).

    Next
    Axiomatic mathematics must have distinctions between systems in order to exist. But the boundaries can't be seen. There is nothing to measure. They are the poster child of belief without evidence. — Treatid


    I don't understand what you're stating here. Could you give an example?
    Philosophim

    The Principle of Explosion

    The Principle of Explosion is why inconsistency is a problem.

    Note: Here's a link to Wikipedia's description of the Principle of Explosion: https://en.wikipedia.org/wiki/Principle_of_explosion

    It is in plain English and doesn't require a mathematics background to understand. Anyone on these forums is likely to find it quite straightforward.

    Note 2: An inconsistency occurs when two statements contradict each other or a single statement contradicts itself (c.f. Liar's Paradox).


    The Principle of Explosion shows how a single contradiction in a system means that we can contradict every possible statement in that system.

    In Axiomatic Mathematics, statements within a consistent system are proven. Statements in an inconsistent system are garbage.

    For Axiomatic Mathematics to work there needs to be statements that are not inconsistent.

    The Principle of Explosion says it applies to any system that contains an inconsistency but does not otherwise explicitly define what a system is or what the scope of a system is.

    However, we can examine the mechanism of explosion.

    The Wikipedia example is written in English and uses contradiction phrased using English sentences.

    Can we apply the Principle of Explosion to other English sentences? Clearly we can.

    Can we apply it to all other English sentences? Yes.

    Can we apply it to German sentences? Well... yes...

    The explosion relies on the initial contradiction. The subsequent statements don't even have to be in a specific language or even meaningful.

    The mechanism of The Principle of Explosion means that given a single inconsistency, every single statement that could possibly be made can be shown to be inconsistent.

    Somebody forgot to include a stop function.

    Check for yourself

    The Principle of Explosion says that given a single contradiction every possible statement in every possible language is inconsistent.

    And you can see the process for yourself. Axiomatic Mathematics has gone bye bye and you don't need a mathematics degree to understand why.

    This... is significant. Axiomatic Mathematics is a Grand Illusion that never existed.

    Don't take my word for it. Examine The Principle of Explosion for yourself. Determine for yourself that the allusion to 'system' is a red herring. The Principle of Explosion necessarily applies to every conceivable statement.

    Try re-writing The Principle of Explosion so it can be constrained. See if you can save Axiomatic Mathematics.

    Hint: This isn't due to a typo or some quirk of an informal language. This is the result of mathematicians being so desperate to preserve Axiomatic Mathematics that they very carefully did not fully examine one of the principles they relied upon.

    Extraordinary Claims require extraordinary evidence

    I have made an extraordinary claim: "Axiomatic Mathematics doesn't exist".

    I have provided evidence: The Principle of Explosion isn't constrained.

    You don't need to take anyone else's word on this. This is within your ability to determine for yourself.

    I will tell you that I'm not misrepresenting anything but you really don't need to trust me. Just read The Principle of Explosion and understand how it works.
  • Philosophim
    2.4k
    The Principle of Explosion is why inconsistency is a problem.Treatid

    Sure, but you never pointed out the inconsistency that destroys Axiomatic math.
  • Kizzy
    99
    While the belief in an objective reality distinct from our subjective perceptions is widespread - it is a belief without evidence.Treatid

    Evidence based belief sounds weak to me...like its reasonable behavior of a sheep or a follower. I don't question faith unless I have to...

    Belief without evidence is valid enough for you question or call out but when is it good enough to accept? Are you aware of the boundary or line that seems to exist for you (disregard others at this point, and do that [answer boundary question] with honesty FOR best results) to make that call of judgement? DO you trust your judgement? Like when will the evidence be good enough for you to accept? If one just has to present and prove to YOU, maybe it is actually more doable than you imagine...thats never been my intention, I will explain why I shared the video later...I am just rolling with the imagination and your interpretation for now because this is fun. What I am saying here is fresh off the mind, what I will explain later I have had time to stew on.... I have many thoughts and lean typically defending solipsism because I feel its wrongfully understood, I agree with your stance on a few things here in the post....upon of course having my own, which i have shared with a few...I will get to that at a later time, as I am in the process of major organization of gathered notes and ideas from the past few years, its hard and a lot of work...

    Evidence based belief just sounds off to me. Evidence cant be the base to build belief from...

    What if the evidence is real and exists but just hasnt been put together or even noticed/seen yet through the right lens that has the ability (that doesnt exist yet) that can see the big picture for what it actually is outside of us, because we are in it now (thats why we cant know, yet)


    Challenge Time

    If you can describe a static object you will have shown that I'm wrong and that I don't know what I'm talking about.

    The two main arguments I'm going to fall back on will be:

    A. You haven't actually described anything. "Objects are not relationships" is not a description of an Object.

    B. What you have actually described is relationships. My default position is that if you manage to describe something it must have actually been a (set of) relationsips in the first place.

    This is, of course, a blatant attempt to get you to engage with the ideas of what language is capable of and what it isn't capable of.

    Can you describe something that has no similarities to any of your previous experiences?
    Treatid
    Hm not sure - because no base but a solid library exists...

    How can I if it is happening in the time that has yet to come, and in the time that I will not be able to compare to previous experiences but only know what it actually was that was happening now? Ha!

    I am with you though I think, a picture of the final version cant exist because it isnt real yet....i dont reject solipsism at all by the way, I lean in certain directions...I blame the wind!

    Confirmed from/in my personal notes (few years old, subject to change and some has):

    "CANT MENTION BASE WITHOUT KNOWLEDGE ON SURFACE ITS BASED UPON, IS IT FIXED, LINKED, CONSTRAINED? IS THE BASE "FIXED" NOT NECESSARILY THE SURFACE OR FOUNDATION? OR IS THE BASE THE FOUNDATION AND YOU ARE FORGETTING IMPORTANT MAYBE NOT IMPORTANT BUT RELEVANT DETAILS THAT EFFECT OVERALL USE OR PURPOSE OF FUNCTION-ABILITY AS A DESIGN IN ACTION....IS IT A DESIGN ANY MORE OR A VISION AGAIN? WAS THE DESIGN THE EVIDENCE? AND THE PRODUCT, AS IS, NOT WHAT IT CAN BECOME, THE VISION TURNED INTO MATTER....DESIGNERS ARE INTELLIGENT AND ALSO MOST FIT TO PRODUCE, TRAIN, TEACH OTHERS HOW TO BRAINSTORM AND DRAFT DESIGNS AT BASIC TO EXPERT LEVELS...PROBLEMS ARE THE BUILDING PART, CHEAP MATERIALS OR PAYOUTS/RATES FOR WORKERS, BAD QUALITY, BAD PRODUCT, BAD DESIGNER.....GOOD VISION? LACK OF INTELLIGIBLE INFO TO PROCEED DESIGNING FROM A BASELESS IDEA, BUT NOT USELEESS MAYBE VERY USEFUL OR OF USE, JUST WITH THE RIGHT QUALIFIED HELP OF TRAINED EXPERIENCED CREDIBLE PROFESSIONALS OR EXPERTS IN FIELD SPECIFIC TO DESIGN TECHNIQUES/ PROCESS AS A WHOLEEEE NOT JUST THE DRAFTED PLANS OR SKETCHES FROM FIRST VISION...BELIEVE IN THE VISION, THROUGH A GOOD LENS TO CONTRIBUTE TO THE DESIGN...INTELLIGENT DESIGN EXISTS BC THE DESIGNERS HAVE THE INTEL AKA KNOWLEDGE TO SHOW, AND EXPERIENCE ACHIEVEMENTS ESTABLISHED BIZZ OR NAME MADE OF ONES SELF...HAPPY CUSTOMERS/COWORKERS/PARTNERS....REFERENCE, REVIEWS, PROOF....WHATS THE GOAL FROM THE VISION VICE VERSA"

    More notes in the same document (from the same day)

    "PHILOSOPHY FORUM 4/11/23 425 AM****additions 9/12/23 530 pm

    -instead of unlikely becoming likely or making the unlikely, likely....its a mechanism that makes the unlikely, possible or real (instead of likely, its actually happening...when did the process begin? Does that moment matter? Maybe not, everything that needs to does..when its supposed to...and " it "happens at the same time to our human realities, the pace at which we actually understand the full truth for what it was worth and how much energy was given and received and recycled and wasted before it was grasped as "something else" from OG, but its "final state" --what is the final version of anything minus time as a constraint...(family legacy, heir, estate, buisness passed down--despite if it happens to be in the right hands from the wrong people, the wrong hands of the only people, (which seems right to the two in relation (son and father or grandfather to only grandchild, parents dead or not in the will for person reasons) [not important atm]) or [ IN IMAGINATION SEEING THE VISION THRU AND THRU....151PM 4/11/23]
    OR in the wrong hands from the right people to the right person who was influenced by the wrong hands, and vice versa all circumstances or scenerios....CAN THE FINAL VERSION ALSO BE THE FIRST VERSION OR AS TIME CONTINUES THE VERSION IS UPDATED WITH THE STANDARDS OR UNDERSTANDING OF STANDARDS BY PEOPLE IN A SOCIETY? Like...the change that may occur, to get to the "final version" is that just simplifying as much as possible to get to its "first version" state, and how relevant is time as speed of successful use of an object??? (something performing is function because its working properly by your hand aiding in its cause to an end (of function and object made to do that work) is in action...one act using your body (like motor memory, riding a bike, operating the fork lift at work bc you do it everyday for a prolonged time-good experience and confidently acting, while still deciding on a way to maneuver to a safe or comfortable "enough" "landing place" -- a break point, a breather, walk away and come back after working on something for too long alone with no feelings of accomplishing "useful or up to standard results, satisfactory to your likeness of work output despite actually putting in NEW EFFORT OR TACTICS OR ROUTES TO GET TO THAT "GOAL" which is more a task, required by obligation or contract or work or school to proceed or receive acknowledgment in career or as a relevant source of truth .....as a matter NOT OF FACT, BUT as the matter in fact of a truth, with and within self, moving with and inside the world as we know it, as we think we know it, what others think they know and what you think you know others think and other facts of these truths....
    lets just say for eg....[a breather after stressful presentation...the anxiety leading up to an event and the relief point when its done and went either better or worse but not as expected....a surprise and reminder to not doubt too long]
    ....and the purpose intended to serve (goal, intent) is both Cause and Effect..."

    END OF NOTES [Shared]

    EDIT:

    TAKE TWO 6/17 135AM

    Can you describe something that has no similarities to any of your previous experiences?Treatid
    Maybe death? How about a near death experience, describing what is happening when a person thinks or actually thinks and IS dying but lives to share the experience...That to me, seems like foreign territory- meaning not a place I can speak from....dying is like nothing that I have experienced before. (unless of course, i have--unless i have my own NDE or actually die how can these descriptions or personal accounts amount to anything except maybe perhaps at best, hope for another?

    Yeah yeah, I cant describe it personally but people still attempt to and documented attempts help credibility not evidence only in the similarity of others that also experienced near death...Of course until i die, i can only these take stories for what they are (again, i am not questioning faith based belief PERIOD and that doesnt mean I wont straight up reject the accounts, I can enjoy a good story and go along for the sake of fun and nothing else....

    Assuming they are really describing it as it happened in reality(how they describe it will be telling and the basis of my judgement, should i believe this person or not-why (reasons to doubt may exist and at times be the right option to act on, instead of giving the benefit of the doubt (which still isnt believing in them, but allowing or entertaining the idea for sake of what? (insert-blank) [amusement? how cruel lol]) and thats IF I decide to believe them beyond all doubt...the option still occurs to me, time presents where I could chose to immediately ignore claims/descriptions, based on my trusted judgement alone not because my belief system is opposing theirs (i cant know their system at large yet at this point of a judgement call- required to proceed in any reality (solipsistic state or not, objective or not)

    I dont have interest in these accounts of NDE, as of now nor have I done any research into these accounts...I was just thinking about the question a bit more before I am on to the next....

    Some people might find comfort in hearing these NDE experiences while others will and can remain skeptical (both reasonably fine by me...im sure and unbothered by both happening at once) I am not losing sleep over this, I have been sleeping on it...
  • Kizzy
    99
    If you were to experience something without any precedent you wouldn't be able to convey your experience.

    "It was like nothing you have ever seen before."
    Treatid
    right, its who is holding the value and using the understanding. self is capable of given the current experience of direct reality (external world-the illusion is too clear (if it is an illusion, i think illusions are to be unclear or blurry at best...unreliable [like what you said above, "nothing like ive experienced before"], until one can focus...provoking the brain, thinking a thought is forced upon brain by us... (goal, "why is the illusion scary at first experience and how does awareness of illusion make them any less real or less scary? "why did that illusion stand out, what made it/is intriguing or interesting in perceiving it?"] Maybe because its unfamiliar territory and the brain cant be stopped...the body wants to stop once it realizes it has to break from usual mindless actions and become real...put to the test. revalue, replaying, remembering, recreating...reusing, refuting, realizing...it isnt what it seems, but on purpose..these illlusions arent just in reality as illusions, but when we see something full circle we call our past beliefs illusions but they cant be that if we are calling them that, awareness of an illusion..distinct difference in grasping what was "normal" or real, is altered..not easily adapted, goes back to old way of seeing, illusion was temporary or self induced (drugs, alcohol, lacking sleep, etc) we can blame or point to answers of how, why we saw an "illusion" based on definitions on paper, and plans can be put in place also from paper but when its show time... how is it not avoidable [tricking the brain] from there or prevented [with others, medical assistance or therapy or under doctor supervision or observation, being monitored makes us feel safe here maybe?

    What if i wanted "illusions" to distract me and purposefully induced illusions to lose track of what was known of reality...the distance from that place(reality as we know it) to the new place (illusion induced reality as I know it, but i chose to do this...so my beliefs and percepts are off to begin with...delusional idea....But what if you had no choice and experienced NDE from a freak accident that occurred in day to day life (car accident or whatever), thats shocking the body and brain to the core, resetting everything we thought we knew, reality check, aims to not take life for granted or excited to die (because the NDE was to good to resist for long)...It becomes delusional believing the illusion when aware of what is the illusion, the choice -- the fact its a choice, (im not judging, i kind of dig it actually) one might be willing to live by/with/in shows that reconsidering all we know come from quick moments, seemingly that flash before our eyes...

    so,the fact that the time may exist for that choice to be made and of the choices some people opt for death via suicide or self destruction...the choice to no longer experience reality as you once did (when physically, change happens because time, growth or aging is inevitable, but maybe the change didnt occur (or change was not clocked or measured because the distance is out of reach) in mindset at a pace that ought to balance self out. Claiming to have had a knowing from a young age (too young if you ask me) that they cant handle the burden of life, chose to follow through with removing the self from its unfair ways (too bad this is not true (bad judgement), because what if fairness can exist but they didnt get to see it through, time was taken and put else where (how much force, direction?)....that type of knowing, I have to admit is an act of strength in itself (that see's self as weak, contradicts the reason for act of suicide considered by me to be an *excuse to act), BUT with courage, and confidence (seemingly) though arguably selfish depending on who you ask... (consequences of others and how they'll deal with their life after death are not the worry of the person who committed suicide and is dead, but becomes of anothers (even though it (they) might have always been a worry for them anyways) UNLESS they (who becomes dead by suicide) was a true believer of solipsism and while in the state/phase that is solips. some authority is using the space in mind, and family/friends who mourn the loss may not be considered worthy enough for after thought at this time, consequences dont matter here (this state or phase)....no family no friends no neighbors, is no one in their picture? solipsism works out for them then and makes this easy i guess and is that alone what justifies and makes them feel better about their choice? I dont know about that... but those are tricks the mind can play as well...tricking self to believing something not actually happening the way others may see,ex. "i dread going to work tomorrow but im going in with a positive mindset otherwise it might be worse than expected in the first place, trick the brain into thinking we are excited for work and will make the best of the day there"

    Someone will find the body (or maybe no one is looking) and brain (whats left)... what is left of your mind from the brain which has shut down officially...you are now not even your mind, you are a dead body with nothing to be remembered by except belongings which can then be used to paint a picture or tell a story (true to your life or retold).

    Mind when it is shut down is now just a limp impression riddled brain, just of another human body judged for what its NOW worth when uncovered and defined and identified, its not up to us now to determine that...what it was worth at the end of the life...memory is preserved and can be maybe unchanged given the constraints that cause change?

    What do i know anyways???

    What can i remember forever, but still need to be reminded with every breath and forgotten by the next step...on repeat. What if forgetting is the goal, what is having a "good" memory going to do for me besides remind..what if the memory was placed at a certain distance on purpose? [235 am END]
  • Treatid
    22
    Evidence based belief sounds weak to me...like its reasonable behavior of a sheep or a follower. I don't question faith unless I have to...Kizzy

    Your posts read like stream of thought. In your second post, particularly, you seem to be riffing on the ideas of existence, hallucination and nihilism.

    The primary relationship between your written ideas appears to be temporal.

    I think knowledge/understanding is directly correlated with the network of relationships we have between ideas. Any given idea primarily exists as its relationship to all the other ideas. The shape of those relationships is where we perceive meaning and significance.

    By presenting your ideas as a single, chronological list of thoughts you strip away much of the rich interplay of concepts.

    I suspect that given a long enough list of thoughts it would be possible to statistically analyse the frequency and position of ideas to get a sense of your internal structure of relationships.

    As such, I recognise what you are saying as genuine reflections of what you think and believe.

    However, I'm not going to do statistical analyses on your stream of consciousness. Partly because no-one has time to do that but mainly because the process of creating structure on the page out of structure in the mind is an essential component of dialogue.

    The effort to represent our internal shapes in an external format is part of learning and growing.

    Even if no-one reads what we write, we learn from the process of structuring our writing. By examining and considering our thoughts as we express them we can change our ideas even before we engage with other people.

    Stream of consciousness can be a useful insight into our own thoughts but as a form of communication with other people it lacks an essential element of self-reflection.

    Next

    The Principle of Explosion is why inconsistency is a problem. — Treatid


    Sure, but you never pointed out the inconsistency that destroys Axiomatic math.
    Philosophim

    As a demonstration of the principle, consider two contradictory statements—"All lemons are yellow" and "Not all lemons are yellow"—and suppose that both are true. If that is the case, anything can be proven, e.g., the assertion that "unicorns exist", by using the following argument:

    1. We know that "Not all lemons are yellow", as it has been assumed to be true.
    2. We know that "All lemons are yellow", as it has been assumed to be true.
    3. Therefore, the two-part statement "All lemons are yellow or unicorns exist" must also be true, since the first part of the statement ("All lemons are yellow") has already been assumed, and the use of "or" means that if even one part of the statement is true, the statement as a whole must be true as well.
    4. However, since we also know that "Not all lemons are yellow" (as this has been assumed), the first part is false, and hence the second part must be true to ensure the two-part statement to be true, i.e., unicorns exist (this inference is known as the Disjunctive syllogism).
    5. The procedure may be repeated to prove that unicorns do not exist (hence proving an additional contradiction where unicorns do and do not exist), as well as any other well-formed formula. Thus, there is an explosion of true statements.
    — Wikipedia

    Here we have the contradicting statements "All lemons are yellow" and "Not all lemons are yellow" proving that "unicorns exist".

    Exactly the same process can be used to prove "unicorns don't exist".

    We have a free proof machine. Plug in any sentence and we can prove that sentence is true.

    If everything is provable, including the direct contradiction of everything; then proof is meaningless.

    The trouble is that everything is provable. It is right there. We can plug in any sentence whatsoever and prove that sentence to be true.

    The Principle of Explosion works for any sentence. Not "any sentence in a given system", not "the particular sentence I happen to feed into it".

    The Principle of Explosion applies to every single sentence in every single language real and imagined.

    You ask me to point to the specific inconsistency that destroys Axiomatic Mathematics...

    You test my ability to take all posts in good faith.

    The Principle of Explosions works with any contradiction. The contradiction doesn't have to be part of a particular Axiomatic System.

    You can see how it works. Any contradiction anywhere, just once, means that every sentence and its contradiction are proven to be true,

    Are you suggesting that the contradiction built into this example doesn't apply because it is made up?

    Given that the made up contradiction clearly and obviously applies to The Principle of Explosion because we can see it being applied right here in this example, I'm genuinely confused as to what you are expecting.

    Threshold

    The threshold here is: "is it possible to state a contradiction".

    An entirely consistent system doesn't contain the concept of contradiction. It is (in principle) impossible for a consistent system to describe a contradiction.

    If it is possible to describe a contradiction it is already far, far too late.

    The only way to have a consistent system according to The Principle of Explosion is if there are no contradictions anywhere, ever.

    Just the possibility of one single contradiction anywhere in all of time and space doesn't just kill Axiomatic Mathematics, it removes it from existence.

    As soon as The Principle of Explosion was stated it was Game over man, game Over!
  • Philosophim
    2.4k
    Here we have the contradicting statements "All lemons are yellow" and "Not all lemons are yellow" proving that "unicorns exist".Treatid

    I think you have a severe misunderstanding. First, an assumption that two competing premises are true does not prove they are both true. In this case its clear that all lemons are not yellow is true, while "All lemons are yellow," is false.

    Let me indicate what the Principle of Explosions is really indicating.

    Lets say that I have the following proof.

    A = A
    A -> B
    B -> C

    If all are true, then this is proven. However, lets say that we discover that A -> !B. We now have a contradiction that invalidates this particular proof. But that does not invalidate logic.

    To invalidate logic, we would need to discover that both A -> B and A -> !B and both are true. Again, not in the metaphorical sense but in the literal. At that, a fundamental of logic, that something cannot be proven to be both true and false at the same time would be invalid and result in the Principle of Explosion you're talking about.

    1. We know that "Not all lemons are yellow", as it has been assumed to be true.
    2. We know that "All lemons are yellow", as it has been assumed to be true.
    — Wikipedia

    To your example, you should not be able to see why this doesn't cause an Explosion. The assumption of truth is not the proof of truth. Often times in logic, assumptions are made and later proven false. This is actually a logical method of proof called Reductio Ad Absurdum. https://en.wikipedia.org/wiki/Reductio_ad_absurdum

    To prove a contradiction exists, you must demonstrate that something both literally does, and does not exist at the same time. So far, no one has been able to demonstrate this within Axiomatic mathematics, so the logic stands.

    You ask me to point to the specific inconsistency that destroys Axiomatic Mathematics...

    You test my ability to take all posts in good faith.
    Treatid

    My intention was only to get clarification. A question asking you to clarify your stance is what good faith argumentation does. This helped me to see what you were thinking about, and allowed me to address the argument to your premises, and not my own. I wanted to avoid a "Straw man fallacy", or an instance in which a person invents or points to a premise the OP is not talking about, and makes the argument about that.
  • Kizzy
    99
    As such, I recognise what you are saying as genuine reflections of what you think and believe.

    However, I'm not going to do statistical analyses on your stream of consciousness. Partly because no-one has time to do that but mainly because the process of creating structure on the page out of structure in the mind is an essential component of dialogue.

    The effort to represent our internal shapes in an external format is part of learning and growing.

    Even if no-one reads what we write, we learn from the process of structuring our writing. By examining and considering our thoughts as we express them we can change our ideas even before we engage with other people.

    Stream of consciousness can be a useful insight into our own thoughts but as a form of communication with other people it lacks an essential element of self-reflection.
    Treatid
    Treatid, I wouldnt task my worst enemy with such a job of analyzing my BS...maybe because its a bit revealing but it is indeed a hot flaming mess burrowed yet not letting the void keep anyone stuck. I am fired up in this void, even alone...I light up with any glimpse of acknowledgment, so thanks for that at least.

    I dont have to thank you, I know and its not that you have done anything at this point that wasnt anticipated-- its common decency, to me. [who is it I could be thanking and who is it that takes this for only that?]

    The progression is slowly but surely still unfolding for me in real time. I simply felt moved by the thread and wanted to share something I thought (in that moment) was connected to previous thoughts I have had years prior when I was first introduced to the video. It is clear that I didnt add anything specific regarding my goals, intentions, or motive regarding your post. I hope sharing my self help streams in this manner isnt too distracting and I applaud the way you were able to expand on more points using the interpretation you got from me...that tells me you are particular maybe detail oriented, patient, and busy... It also tells me you are passionate about your work here and time is critical to get certain things accomplished. Only we can know how to spread ourselves and what that means for positive forward motion certain alignments are required in the body, mind, and the will; that is attached itself somehow now to the goal...if there is a will, there is a way! Basic and ringing true to me still.

    ANYWAYS, what YOU happen to see is actually a true (and awfully accurate) understanding and by that ability you harness valuable conversation or exchange. Reciprocity. Thank you for sharing, thinking, communicating, learning, and teaching...The clarity is so important, especially for a delinquent fire rat like myself rattling around the pipes after deep burrowing phase. I appreciate the time you have put in and spent here...and not to mention what it took to get to this point, that work deserves a round as well. Bravo!

    P.s. I am not forcing anything by sharing my notes or personal reflections, I do not expect nor intend to add to the work load of others in my attempts at sharing such common yet complex words of expression...I expect people to just get or assume they already know what I do and thats just incredible silly and wrong of me. "not everyone thinks the way you do, kizzy" "you have an answer for everything, kizzy" said in sarcasm to me from my own father. Yet here I go doing that again. I will work on it, I am reminded about the lack luster in such attempts from the readers perspective. I am learning, and will make up time with quality and passionate interest in learning how people learn.

    I am happy that you acknowledge (very least besides straight ignoring) what I have been true in, in my efforts to understand myself, others, and even some times ability to notice/recognize patterns that I feel intuitively might be a piece to a puzzle of nothing I know of in the moment...I dont know what I am trying to say yet, but this process I am in obviously needs organized and focused clarity to communicate these ideas into a coherent "thing". But thats the thing....i have no goal on the forum, agenda, I dont know what thing I am even getting at but i just feel like I eventually may. I also am passionate on this path and am driven through a spark of interest....I like like-mindedness and crave deeper understanding in knowledge--human nature at large is under the scope.

    This is just one part, of a connected whole. This part can be designed though to not be "the part serving a function or purpose by just playing a role" what if the part is designed to be a whole in itself and what if that part is designed to not just work in a position of a whole, but designed to work in many different wholes as a part. Im thinking like interchanging parts of wholes... Similar yet so familiar that its doable, its good enough, it functions. The consumption, the hand is just that and it needs > wants more guidance and direction, i feel.

    I am aware of the loss that comes from my style when it comes to reaching people and communicating properly. But this is a forum and even though proper communication should be had face to face, via zoom call or phone call is even quicker than back and forth commenting here. A lot of over explaining and talking over/straight disregard. Thats not a conversation I would be participating in real life, I would walk away from such a heartless fool. I do not need anything from this place that I cant get in real life...I just sought those who have similar interests as I...philosophical discussion. I am self taught and dont know a lot of the terms in this ever growing tree of branches and branches...the fruits of the labor are becoming evasive to the trees nature...the growth stops and loops back into the same ideas. Forward thinking and limitless design is what I am interested by.

    The conversations here seem to pick up and take off with no immediate back and forth at times, I am fine with that...actually kind of prefer it. But a real person to person communication in daily life is no easier and confusion still occurs. I dont get that....unless confusion occurs simple from not caring in the moment whats about to come and expecting different outcomes...self-awareness degree is/are/shall be revealed at.

    It seems like just communicating in general is what people want to do, those who just talk to talk, they like the sound of their own voice. Some just need someone to talk to, thats lame but fine...Perhaps, it is true that I just dont care to do that right now...I have people who are more than willing to analyze my BS... I am surrounded by that I can practice with. But I dont need to practice communicating, I need to decide what the hell I want to do. I am burrowed and expressing outwardly here from there for a reason... its clearly possible from your last reply to me alone that people pick and chose what to read and what to ignore. Fair, I do too. That is telling though in itself. What and where we find ourselves over and over again. Back for a reason...back to the drawing board it is! (exciting, but where is square one from here?)

    Time is not consuming just constraining and while I agree with that when you said it--but whatever that excuse may be, even though its valid to use time as an excuse, [as it constrains our and limits what is possible to get done in a 24 hr day]...there is always tomorrow, I keep telling myself the professional procrastinator!

    Over all, I still think what I was doing can be seen even if not fully reading into every word (ugh the agony that must be for other readers here, wah wahhhh :cry: well, boo hoo! cry all you want, baby...victim mindsets are triggering for me, i digress [barely])

    TO BE ABLE TO call out what I am doing, as you have made it apparent that it is doable...is not hard but I think whats more interesting then the fact I got a response, is the fact you have no opinion on my answer to the question (didnt read it, fine but why comment if you read into it enough to come up with a reply telling me what i am and what is going on. I appreciate the guidance you offered, but feel as if I you see what I am and doing almost like one can see a roomba stuck behind or caught behind a door still giving it the best efforts it can to do what its suppose to...clean. I am not cleaning if I am being messy, so i can now see the relation you may have seen and I am laughing while typing this.

    Did you think I wanted to be analyzed instead of just responded to? Thats wildly embarrassing for me...I hate that people may think that...but what will i do about it? I at least have something to be organized in the first place, a lot of folks are ... lacking in areas I most certainly can make up for, if allowed that grace in attempting to make the best version shine. I cant shine without this elbow grease....(my hand and back and eyes are strained but the will is strong and mighty, thats a sacrifice I am willing to offer.

    If a statistical analysis is what must happen for you or anyone to see, hear, understand, follow me...I better find a trust worthy person to do that and will when the time is right. I dont know how much of that is true nor do I care right now. When i gave my reflection the second time, i was answering a question you asked in another comment in the privacy of my mind and that is what was shown publicly, right in your face and others...a different way of looking at the question, really going there in my mind and typing what comes up as it does. I am reading my reflections over and over again all the time and seeing things I didnt notice when I first typed them...its fantastic and interesting to me. Why? I dont know.

    It is what it is...

    I will take the attention you have brought to me and use it as a reminder to self-- to ask this question before posting a contribution next time: do I really want to go full kizzy mode on these people? I think when I answer "no" that is because I want to actually communicate with others. I think when I answer "yes" I have something I want to get out of my mind and into reality and dont need to communicate because I am just now articulating it and communicating with myself. The words may be fresh to you, that only means its just as fresh to me...I post my reflections in the comments on topics that I find relevant to my life, current intel and knowledge gathered thus far and implement that to the new ideas presented to me via forum..its all just to see if its possible for me to ground or base my stance upon anything here. I would understand if I overly shared on every post, forcing my personal pages upon faces that asked nothing of me... If I had thousands of comments and unwarranted engagement I could see how that becomes annoying. But i specifically comment and engage upon things for an unknown,yet important enough reason TO ME...i risk sacrificing myself and work and time and character just to make something clearer for myself...that is all I can do right now..the fact that others find these topics important is just fuel..

    By only letting me know or others that you can see (not blind) what is going on at the surface of what you read off the screen IS GOOD and progress for me personally...So while I am pleased to get a reply at all, I understand why I dont get them often. I'm fine with that and accept the results i have sown. I have more choices to make now, like I needed another...

    I am bothered when I am but free from a will to worry for long. Trust. My changing thoughts surrounding these deeper though provoking, philosophical topics and terms (which I have been not exposed to in my day to day life, I dont have people around me who want to get into things to the depths that I enjoy being aware of and in...I work with old school type engineers who tell me to drop out of school (lol) and I run with the party people, amongst dear family and friends I hold close to heart...but where I come from, people avoid philosophical conversation, i cant ask the questions I want without being a bother it feels like sometimes....In the deep end I float and sink to depths of darkness and uncertainty and while fear is present it feels bearable. Is that why would I stay down there for this long? Even when all the fun is seemingly being had up at the surface???? Well I do surface but thats because I need a breather! Yes, that can be my excuse to also chose to have some fun while breathing.... A chance to see the good from the bad and navigate freely, how can I be anything but grateful?

    To wrap it up (didnt think I would go this far replying to you pointing out nothing except the obvious but kindly replying--decent of you) sometimes I just feel moved to share something I see might have an inkling of relevance, and yes like you said the important part in that is what people can learn from doing things this way. I have learned many things from doing this alone in this one place, the philosophy forum. What a time and place!

    Enough space taking in this thread for me, excuse me while you scroll on past. I am sorry that I am not sorry for that delay I may have caused...time is of the essence!

    But time will mind me because I have paid mind to it... pay it mind, or dont!

    Thanks!
  • Treatid
    22
    I think you have a severe misunderstanding.Philosophim

    Right back at ya, big fellow (in a friendly, affectionate and non-gender assumptive manner).

    Common terms

    Formal and informal:

    Logic existed before mathematics formalised it.

    Natural languages (such as English) evolve over time. There are no explicit rules about what can and can't be done with natural languages. There are conventions, but even these evolve with the language and are more what you'd call 'guidelines' than actual rules.

    Informal logic is an aspect of natural languages and has no strict rules.

    Axiomatic Mathematics is an attempt to improve on natural languages by providing strict rules that are applied consistently. Mathematics refers to these strict systems as formal systems (as opposed to natural languages being informal systems).

    Formal Logic isn't the same as informal Logic.

    It is like films that are "inspired by..." If you look closely you can see hints of the original story - but by and large they went in their own direction.

    The mechanism of Axiomatic Mathematics (including formal Logic)

    Part 1

    An axiomatic system has a set of axioms (assumptions).

    These axioms can be anything. Absolutely anything. There is no constraint on what axioms you choose.

    "Unicorns exist" is a valid axiom. "All lemons are yellow" is a valid axiom. "BlubBlubBlub" is a valid axiom.

    The axioms determine what the next valid statements in the Axiomatic System will be. In the case of "BlubBlubBlub" it would be convenient to have another axiom in the set which tells us what rules apply to "BlubBlubBlub" to construct the next statement.

    Part 2

    The rules that are explicitly or implicitly part of the axioms are applied to the initial set of axioms to get subsequent statements that are part of the axiomatic system.

    This is repeated with subsequent stages include any new statements that have been generated.

    The process ends when no new statements can be generated or when two statements contradict each other.

    Some Axiomatic Systems do not terminate.

    E.g.
    Valid, non terminating axiomatic system:
    {
    "A"
    "Append "A""
    }

    This system contains "A", "AA", "AAA", "AAAA", "AAAAA", ...

    The statements do not appear to contradict other statements therefore this is a valid axiomatic system and each statement within the system is proven.

    Proof

    A proof only exists within a specific axiomatic system.

    If you change the initial axioms you have changed the axiomatic system - it is now a different, distinct, axiomatic system. A statement is defined by its context. The same words can appear in two different axiomatic systems and be proven in one system and inconsistent in another system.

    Proof = "consistent within an axiomatic system"

    Your belief in the reality or otherwise of a given axiom is irrelevant. If "All lemons are yellow" is a statement within a consistent axiomatic system then "All lemons are yellow" is proven (within that axiomatic system).

    Loose ends

    Axiomatic Mathematics works on the basis of anything not disproven is proven.

    An axiomatic system is inconsistent if it contains a contradiction. It is almost always impossible to show that an axiomatic system is consistent.

    One of the problems is that axiomatic systems actually include all statements that are valid according to the rules of the axioms. Some of these statements cannot be found by iterating on existing statements.

    Most significant axiomatic proofs rely on axiomatic systems that cannot be proven to be consistent. They are believed to be consistent. Mathematicians have tried hard to find contradictions within these systems. The longer a system goes without a contradiction being demonstrated, the more confidence that system is given as a consistent system

    Important features to note

    There are no subjective judgements on what is real.

    The determination of proof is entirely based on consistent or not consistent.

    There are no arguments over whether the axioms are justified, real, true, meaningful, significant or anything else.

    If a set of axioms leads to a consistent system; those axioms are proven.

    As much as I clown on Axiomatic Mathematics; mathematicians went to serious efforts to build a system free of bias.

    Your personal opinion on what is true is irrelevant. What can you show?

    It is a huge credit to mathematicians that they tried incredibly hard to make their assumptions explicit and their arguments falsifiable.

    You pointing at a blue lemon does not disprove "all lemons are yellow" as far as Axiomatic Mathematics is concerned.

    The Blue Lemon

    What is blue? Define blue. Is blue the same category as yellow? can a lemon be both blue and yellow?

    Is what you are pointing at a lemon? Is a real lemon equivalent to a conceptual lemon? Are lemons a figment of your imagination?

    Are all lemons identical? How different can one lemon be from another lemon and still be a lemon?

    Do I exist? What is the meaning of existence? And etc, etc and and etc.

    Mathematicians sought a solution that doesn't involve all the mucking around in philosophy forums.

    "Given a set of statements and associated rules; do we arrive at a contradiction? yes or no?"

    Formal (mathematical) logic is not the same thing as informal (English) arguments.
  • Philosophim
    2.4k
    I think you have a severe misunderstanding.
    — Philosophim

    Right back at ya, big fellow (in a friendly, affectionate and non-gender assumptive manner).
    Treatid

    Ha ha! All good! Please assume the same attitude from my end. A good discussion is what these places are for. I respect your view points and approach.

    These axioms can be anything. Absolutely anything. There is no constraint on what axioms you choose.Treatid

    True, but that doesn't mean they cannot later be determined to be true or false within the system. A consistent and complete system is when every statement is proven true or false from the systems axioms without contradiction. So you can introduce an axiom that states, "The moon is made of green cheese," but to be consistent and complete, it can't be contradicted down the road with other information.

    Proof = "consistent within an axiomatic system"

    Your belief in the reality or otherwise of a given axiom is irrelevant. If "All lemons are yellow" is a statement within a consistent axiomatic system then "All lemons are yellow" is proven (within that axiomatic system).
    Treatid

    Its only proven as long as "There are some lemons that are not yellow" is not introduced. Once you introduce a contradiction, then the system is no longer consistent or complete. But finding a contradiction within a specific axiomatic process does not invalidate the system of axiomatic mathematics.

    It is almost always impossible to show that an axiomatic system is consistent.Treatid

    This is a bold claim. Lets say this is true. Does the fact there are more inconsistencies negate the value of those systems which are consistent? For example, 2+2= ? will have far more many wrong answers than right answers. Does that invalidate math?

    One of the problems is that axiomatic systems actually include all statements that are valid according to the rules of the axioms. Some of these statements cannot be found by iterating on existing statements.Treatid

    Then it is not a complete system. Axioms that cannot be determined as true or false from other axioms on the system are called independent axioms. An entire system of independent axioms is considered an independent system (and probably not very useful)

    Most significant axiomatic proofs rely on axiomatic systems that cannot be proven to be consistent. They are believed to be consistent.Treatid

    But they are not consistent. That's induction, not deduction.

    What is blue? Define blue. Is blue the same category as yellow? can a lemon be both blue and yellow?Treatid

    This is an issue of clearly defining your definitions based on context. We are the one's who determine such definitions. But clearly mark them out, and you can use them objectively.

    Are all lemons identical? How different can one lemon be from another lemon and still be a lemon?Treatid

    Once again, that's up to us as definers to set the parameters of the word we're using. Once you do that, you have what you need.
bold
italic
underline
strike
code
quote
ulist
image
url
mention
reveal
youtube
tweet
Add a Comment

Welcome to The Philosophy Forum!

Get involved in philosophical discussions about knowledge, truth, language, consciousness, science, politics, religion, logic and mathematics, art, history, and lots more. No ads, no clutter, and very little agreement — just fascinating conversations.